LSAT and Law School Admissions Forum

Get expert LSAT preparation and law school admissions advice from PowerScore Test Preparation.

 Administrator
PowerScore Staff
  • PowerScore Staff
  • Posts: 8929
  • Joined: Feb 02, 2011
|
#23469
Complete Question Explanation

Weaken-#%. The correct answer choice is (A)


In this stimulus we see a common logical error: in comparing two options, the author presents a single benefit and concludes that the option which includes that benefit is preferable. Here, the author discusses the fact that about 12 new cases of polio arise each year as a result of the use of OPV, whereas a switch to IPV would cut this figure in half. Based on this limited information, the author concludes that it is time to switch to the IPV.

This sounds reasonable, but there might be more to the picture. Since this is a Weaken question, the correct answer choice will almost certainly present either a new detriment associated with IPV, or a new benefit associated with OPV.

Answer choice (A): This is the correct answer choice. As predicted, this correct answer choice introduces a detriment associated with the author-recommended IPV. In recent years there have been no new cases of naturally occurring polio in North America—if the switch were made as the author recommends, this number would increase from zero to "at least a few." Since this choice presents a detriment associated with IPV, it weakens the author's conclusion that we should make the switch.

Answer choice (B): The fact that most of those affected had weakened immune systems plays no role in assessing which vaccine to use, so this answer choice does not weaken the author's argument.

Answer choice (C): The rarity of the disease compared with other hazards is irrelevant; the stimulus is focused on how best to minimize even the very low current numbers.

Answer choice (D): What people currently use in different locations does not play into the author's argument, which concerns the question of which vaccine will produce the best results.

Answer choice (E): If most vaccines carry this same risk, then it is likely that OPV carries the risk as well. This would not weaken the author's argument unless we knew that OPV carried no such risk. Since we are not provided with any information regarding the OPV seizure risk, this answer choice does not weaken the argument and is thus incorrect.
 jm51
  • Posts: 8
  • Joined: Aug 29, 2014
|
#16958
Hi,

I have a really hard time with weakening questions unfortunately. Although I did answer this question correctly, I don't understand why it's correct other than it was the best answer. If there had been a more tempting wrong answer I would have for sure chosen it.

The reason I don't like (A) is that it says "at least a FEW new cases..." which too me sounds like a lot less than the amount of OPV cases per year (12).

So considering this doesn't that still make IPV the better choice?
 David Boyle
PowerScore Staff
  • PowerScore Staff
  • Posts: 836
  • Joined: Jun 07, 2013
|
#16974
jm51 wrote:Hi,

I have a really hard time with weakening questions unfortunately. Although I did answer this question correctly, I don't understand why it's correct other than it was the best answer. If there had been a more tempting wrong answer I would have for sure chosen it.

The reason I don't like (A) is that it says "at least a FEW new cases..." which too me sounds like a lot less than the amount of OPV cases per year (12).

So considering this doesn't that still make IPV the better choice?
Hello jm51,

"Few" is a fairly flexible term. 12 sounds like a "few" to me, in that 100,000 sounds like many. So, all things considered, A is the best answer, since even 13 would be a "few", and 13 is more than 12.

Hope this helps,
David
 Haleyeastham
  • Posts: 33
  • Joined: Aug 03, 2015
|
#19597
I am confused how A is the correct answer- though this does cite a downfall in going with IPV, the argument only claims that IPV would cut vaccination caused polio in half, not polio in general- answer choice A refers to naturally occurring polio rather than the vaccination caused polio that the argument refers to. Please help me understand.

Thanks!
 Steve Stein
PowerScore Staff
  • PowerScore Staff
  • Posts: 1153
  • Joined: Apr 11, 2011
|
#19623
Hi Haley,

That's a good question; the use of numbers and percentages, along with the scientific terminology (OPV vs. IPV, oral vs. inactivated polio vaccines), represent deliberate efforts on the part of the test makers to make such questions more challenging. For simplicity, let's refer to these two vaccines as the O vaccine and the I vaccine.

For several years, there have been no new cases of naturally occurring polio, and about 12 cases that arise from the use of the O vaccine. If we were to switch to the I Vaccine, there would only be about 6 vaccine-caused cases of polio per year.

Based on the fact that the I vaccine causes fewer cases of vaccine-caused polio, the author concludes that it is time to switch to the I vaccine. We should keep in mind, though, that we might not have all the relevant information. This is what we know thus far:

O vaccine: :arrow: vaccine-caused cases of polio: 12. Naturally occurring cases: 0
I vaccine: :arrow: vaccine-caused cases of polio: 6 . Naturally occurring cases: ?

So, the I vaccine causes fewer vaccine-caused cases, but what if it doesn't provide equally effective protection against the naturally occurring variety?

Since the stimulus is followed by a weaken question, the correct answer choice will hurt the case for the vaccine switch—either by providing another benefit associated with sticking with the current vaccine, or by showing some down side to the switch. Correct answer choice (A) provides that down side: At least a few more naturally occurring cases of polio. This certainly weakens the argument for the switch, because now the comparison looks more like this:

O vaccine: Vaccine-caused cases: 12 Naturally occurring cases: 0
I vaccine: Vaccine-caused cases: 6 Naturally occurring cases: at least a few

With this new information, a switch to the I vaccine looks less appealing: sure, it would lead to a decrease in the vaccine-caused cases, but it apparently doesn't protect as well either—at least a few naturally occurring cases would result—maybe more! Either way, this choice certainly weakens the case for the switch.

I hope that's helpful! Please let me know whether this is clear—thanks!

~Steve
 mpoulson
  • Posts: 148
  • Joined: Mar 25, 2016
|
#24897
Hello,

I have read the book explanation and this explanation above. However, I don't understand why B is wrong. If there are a group of kids with preexisting conditions that make them susceptible to polio and that won't change (with the switch to IPV) then it would seem like less of an incentive to move to a different vaccine. My only thought process for this is that answer A actually leads to more cases of polio where B creates an environment where they may or may not continue to occur. Please explain.

- Micah
 Clay Cooper
PowerScore Staff
  • PowerScore Staff
  • Posts: 241
  • Joined: Jul 03, 2015
|
#25013
Hi Micah,

Thanks for your question.

I'm a little confused by what exactly you mean but I will take a stab at it.

Answer choice A is right because if it were true, then by trying to eliminate the 12 annual vaccine-caused cases of polio by switching from OPV to IPV, we would actually cause some naturally-occurring polio, which doesn't happen right now. In other words, if A is correct, then switching vaccines, which the stimulus argues in favor of, might very well end up causing more cases of polio than we currrently have.

Answer choice B, even if it is true, does not provide any reason not to switch from OPV to IPV. Imagine that it is true that OPV only really causes polio in kids with undetected immune deficiencies - does that offer us any reason not to switch to IPV? No. Would the same be true of IPV - that is, would IPV also only cause polio in kids with undetected immune deficiencies? We are not told.

In short, answer choice A introduces the possibility that switching from OPV to IPV could result in a net increase of polio cases, and thus it provides a good reason not to switch (and weakens the argument, making it a good answer). Answer choice B, on the other hand, provides no meaningful new information; it really only provides more detail about how excactly OPV causes those 12 cases per year of polio.

I hope that helps! Third time's a charm.
 bli2016
  • Posts: 67
  • Joined: Nov 29, 2016
|
#33398
Hi, this is sort of a question out of curiosity... I understand why A is correct, but if E had explicitly stated that IPV carries a slight risk of inducing seizures in children with neurological diseases and OPV does not, would that have made E a stronger answer than A? Just a hypothetical here. Thanks!
 Charlie Melman
PowerScore Staff
  • PowerScore Staff
  • Posts: 85
  • Joined: Feb 10, 2017
|
#33427
bli2016 wrote:Hi, this is sort of a question out of curiosity... I understand why A is correct, but if E had explicitly stated that IPV carries a slight risk of inducing seizures in children with neurological diseases and OPV does not, would that have made E a stronger answer than A? Just a hypothetical here. Thanks!
Hi bli,

First of all, it's great that you're thinking of ways to rework LR answer choices. That's a great technique for understanding the mechanics of the test and improving your score.

To answer your question, your amendment would not make answer choice (E) stronger than answer choice (A). The reason is that (E) is out of scope. The stimulus never mentions seizure risk or neurological diseases. Maybe it's better, all things considered, to reduce polio risk in children with neurological diseases by x amount at the cost of increasing seizure risk by a "slight" amount. Maybe those diseases are extraordinarily rare. The stimulus never says anything about the topics mentioned in this answer choice, which should be a huge red flag.

Hope this helps!

Get the most out of your LSAT Prep Plus subscription.

Analyze and track your performance with our Testing and Analytics Package.